On 08 Dec 2017, at 00:40, agrayson2...@gmail.com wrote:



On Thursday, December 7, 2017 at 4:44:01 PM UTC, Bruno Marchal wrote:

On 07 Dec 2017, at 00:12, agrays...@gmail.com wrote:



On Wednesday, December 6, 2017 at 4:01:31 PM UTC, Bruno Marchal wrote:

On 05 Dec 2017, at 11:32, agrays...@gmail.com wrote:



On Tuesday, December 5, 2017 at 7:57:57 AM UTC, agrays...@gmail.com wrote:


On Tuesday, December 5, 2017 at 5:32:22 AM UTC, agrays...@gmail.com wrote:


On Tuesday, December 5, 2017 at 3:59:19 AM UTC, agrays...@gmail.com wrote:


On Tuesday, December 5, 2017 at 12:50:54 AM UTC, Bruce wrote:
On 5/12/2017 11:38 am, agrays...@gmail.com wrote:
On Tuesday, December 5, 2017 at 12:26:58 AM UTC, Bruce wrote:
On 5/12/2017 3:15 am, Bruno Marchal wrote:
> On 01 Dec 2017, at 01:49, Bruce Kellett wrote:
>> On 1/12/2017 8:57 am, Bruce Kellett wrote:
>>> On 1/12/2017 4:21 am, Bruno Marchal wrote:
>>>> On 29 Nov 2017, at 23:16, Bruce Kellett wrote:
>>>> On 30/11/2017 2:24 am, Bruno Marchal wrote:
>>>>>> On 29 Nov 2017, at 04:59, Bruce Kellett wrote:
>>>>>>
>>>>>>> I would suggest that there is no such world. Whether a coin
>>>>>>> comes up head or tails on a simple toss is not a quantum event; >>>>>>> it is determined by quite classical laws of physics governing
>>>>>>> initial conditions, air currents and the like.
>>>>>>
>>>>>> It depends. If you shake the coin long enough, the quantum
>>>>>> uncertainties can add up to the point that the toss is a quantum >>>>>> event. With some student we have evaluate this quantitavely (a >>>>>> long time ago) and get that if was enough to shake the coin less >>>>>> than a minute, but more than few seconds ... (Nothing rigorous).
>>>>>
>>>>> That is a misunderstanding of quantum randomness. For the outcome >>>>> of a coin toss to be determined by quantum randomness, we would
>>>>> have to have a single quantum event where the outcome was
>>>>> amplified by decoherence so that it was directly entangled with
>>>>> the way the coin landed. Schematically:
>>>>>
>>>>>  |quantum event>|coin> = (|outcome A> + |outcome B>)|coin>
>>>>>  = (|outcome A>|coin heads> + |outcome B>|coin tails>)
>>>>
>>>> The coin is quantum.
>>>
>>> The coin is classical, consisting of something of the order of 10^22
>>> atoms. Indeterminacy in position as given by the Heisenberg
>>> Uncertainty Principle, is undetectably small.
>>
>> I think it is worth while to put some (approximate) numbers around >> this. The reduced Planck constant, h-bar, is approximately 10^{-27}
>> g.cm^2/s. The Uncertainty Principle is
>>
>>    delta(x)*delta(p) >= h-bar/2.
>>
>> For a coin weighing approximately 10 g and moving at 1 cm/s, the
>> momentum is mv = 10 g.cm/s. Taking the momentum uncertainty to be of >> this order, the uncertainty in position, delta(x) is of the order of >> 10^{-28} cm. A typical atom has a diameter of about 10^{-8} cm, so >> the uncertainty in position is approximately 20 orders of magnitude
>> less than the atomic diameter.
>
> I think that is enough to get the macroscopic superposition, as, like
> I explained, you have to take into account not just the quantum
> indeterminacy, + the classical chaos. You might need to shake for some
> minutes.

You could shake for longer than the age of the universe and you will still not convert quantum uncertainties and classical thermal motions into a macroscopic superposition. Do you know nothing about coherence?
And the fact that coherent phases between the components are what
separates a superposition from a mixture?

Are the phase angles of components of a superposition identical? If so, is this the definition of coherence? TIA, AG

No, why should they be equal. You really do have to learn some basic quantum mechanics, Alan, and stop bothering the list with such questions.

Bruce

Was I bothering the list when I started this thread, and others? I recall from class the answer is NO, because the probabilities are unaffected when taking complex conjugates (ignoring interference), but other comments on Avoid2 for example, when I was a member, indicated otherwise. Also, the poster here you're replying to seemed not to understand as well. Next time take THAT into account. AG

I took a few graduate courses in QM at major US universities and do not recall any discussion about coherent states of superposition when solving the SWE or Dirac's equation. Maybe the universities in Australia do a better job. I suggest you be more tolerant in the future, notwithstanding the burden that your knowledge of these subjects places on your karma. And keep in mind that I am responsible the interactions you are now enjoying. That should count for something. Don't ya think? FWIW, I initially left Avoid2 because of the abuse. AG

All I can recall about the phase factor in QM is that it's arbitrary, insofar as whatever value is chosen doesn't effect the result of Born's rule. I suppose that makes me a moron from the pov of an expert. The implication was, for me, that it can't be explicitly calculated. Hence, my question. AG

To wrap it up, if the phase factor is arbitrary, which is what my professor indicated, it's puzzling how a sum of solutions as a superposition could have fixed relations among those factors to yield a coherent wf. I may not be the brightest bulb in the room when it comes to QM, but a dumb question it is NOT. AG

The overal phase factor is arbitrary, but not the relative one in a superposition. If h is a phase factor,(a complex number ith mudulus = 1),

h(a + b) gives the same prediction than (a + b), but (a +hb) will give different prediction

Bruno

From the pov of Born's rule, the phase factor is arbitrary. That's not my problem. I've been reading the Wiki link Bruce posted and I am still unresolved about this issue. For example, in the double slit the waves emanating from both slits have the same phase angle and the interference depends on the distance of each slit from various points on the screen. But if we choose different phase angles at each slit, wouldn't the predicted interference pattern change? AG

Yes, but only if the phase are indeed different at each slits, I would say. The interference pattern would shift.

I had a confusing and testy response from Bruce on this issue. To recapitulate:

AG> Are the phase angles of components of a superposition identical? If so, is this the definition of coherence? TIA, AG

BK> No, why should they be equal. You really do have to learn some basic quantum mechanics, Alan, and stop bothering the list with such questions.

I might be mistaken, but In the double slit I think the phase angles must be equal to get the interference pattern observed, and if they're different at each slit, we won't get what's observed.

That is correct, if they are different, the interference fringes will be shifted, but the interference is still there, and you still don't know through which slits the particles did go through (it makes non sense, because your consciousness relies on both histories).



And if each component of a superposition with many components has an arbitrary phase angle, I don't see how we get coherent waves. I know this is not an interesting issue for Bruce, but maybe he will clarify the situation. IIRC, on another message list, Roahn, a Ph'D physicist known to Bruce, claimed the phase angles of components of a superposition are equal.

You can renormalize it that way, but being cautious doing that coherently with the whole wave. It is tricky and misleading when doing some arbitrary FAPP transformations, and reading that in term of parallel histories, so in "meta-physics" (where the physicist is described with QM) we have to be cautious.



It would seem so, for if one has a solution of the SWE and assigns a phase angle arbitrarily, and then expands the solution in some basis, I think the basis vectors would inherit the same phase angles. Still studying Bruce's link!

OK.

Bruno



TIA, AG

Bruce

Bruno








Random quantum uncertainties
and thermal motions are not coherent, so cannot form superpositions.


--
You received this message because you are subscribed to the Google Groups "Everything List" group. To unsubscribe from this group and stop receiving emails from it, send an email to everything-li...@googlegroups.com.
To post to this group, send email to everyth...@googlegroups.com.
Visit this group at https://groups.google.com/group/everything-list.
For more options, visit https://groups.google.com/d/optout.

http://iridia.ulb.ac.be/~marchal/




--
You received this message because you are subscribed to the Google Groups "Everything List" group. To unsubscribe from this group and stop receiving emails from it, send an email to everything-li...@googlegroups.com.
To post to this group, send email to everyth...@googlegroups.com.
Visit this group at https://groups.google.com/group/everything-list.
For more options, visit https://groups.google.com/d/optout.

http://iridia.ulb.ac.be/~marchal/




--
You received this message because you are subscribed to the Google Groups "Everything List" group. To unsubscribe from this group and stop receiving emails from it, send an email to everything-list+unsubscr...@googlegroups.com.
To post to this group, send email to everything-list@googlegroups.com.
Visit this group at https://groups.google.com/group/everything-list.
For more options, visit https://groups.google.com/d/optout.

http://iridia.ulb.ac.be/~marchal/



--
You received this message because you are subscribed to the Google Groups 
"Everything List" group.
To unsubscribe from this group and stop receiving emails from it, send an email 
to everything-list+unsubscr...@googlegroups.com.
To post to this group, send email to everything-list@googlegroups.com.
Visit this group at https://groups.google.com/group/everything-list.
For more options, visit https://groups.google.com/d/optout.

Reply via email to